Are you seeking one-on-one college counseling and/or essay support? Limited spots are now available. Click here to learn more.

How to Write the AP Lang Rhetorical Analysis Essay (With Example)

November 27, 2023

how to write AP Lang rhetorical analysis essay example

Feeling intimidated by the AP Lang Rhetorical Analysis Essay? We’re here to help demystify. Whether you’re cramming for the AP Lang exam right now or planning to take the test down the road, we’ve got crucial rubric information, helpful tips, and an essay example to prepare you for the big day. This post will cover 1) What is the AP Lang Rhetorical Analysis Essay? 2) AP Lang Rhetorical Analysis Rubric 3) AP Lang Rhetorical Analysis: Sample Prompt 4) AP Lang Rhetorical Analysis Essay Example 5)AP Lang Rhetorical Analysis Essay Example: Why It Works

What is the AP Lang Rhetorical Analysis Essay?

The AP Lang Rhetorical Analysis Essay is one of three essays included in the written portion of the AP English Exam. The full AP English Exam is 3 hours and 15 minutes long, with the first 60 minutes dedicated to multiple-choice questions. Once you complete the multiple-choice section, you move on to three equally weighted essays that ask you to synthesize, analyze, and interpret texts and develop well-reasoned arguments. The three essays include:

Synthesis essay: You’ll review various pieces of evidence and then write an essay that synthesizes (aka combines and interprets) the evidence and presents a clear argument. Read our write up on How to Write the AP Lang Synthesis Essay here.

Argumentative essay: You’ll take a stance on a specific topic and argue your case.

Rhetorical essay: You’ll read a provided passage, then analyze the author’s rhetorical choices and develop an argument that explains why the author made those rhetorical choices.

AP Lang Rhetorical Analysis Rubric

The AP Lang Rhetorical Analysis Essay is graded on just 3 rubric categories: Thesis, Evidence and Commentary, and Sophistication . At a glance, the rubric categories may seem vague, but AP exam graders are actually looking for very particular things in each category. We’ll break it down with dos and don’ts for each rubric category:

Thesis (0-1 point)

There’s nothing nebulous when it comes to grading AP Lang Rhetorical Analysis Essay thesis. You either have one or you don’t. Including a thesis gets you one point closer to a high score and leaving it out means you miss out on one crucial point. So, what makes a thesis that counts?

  • Make sure your thesis argues something about the author’s rhetorical choices. Making an argument means taking a risk and offering your own interpretation of the provided text. This is an argument that someone else might disagree with.
  • A good test to see if you have a thesis that makes an argument. In your head, add the phrase “I think that…” to the beginning of your thesis. If what follows doesn’t logically flow after that phrase (aka if what follows isn’t something you and only you think), it’s likely you’re not making an argument.
  • Avoid a thesis that merely restates the prompt.
  • Avoid a thesis that summarizes the text but does not make an argument.

Evidence and Commentary (0-4 points)

This rubric category is graded on a scale of 0-4 where 4 is the highest grade. Per the AP Lang Rhetorical Analysis rubric, to get a 4, you’ll want to:

  • Include lots of specific evidence from the text. There is no set golden number of quotes to include, but you’ll want to make sure you’re incorporating more than a couple pieces of evidence that support your argument about the author’s rhetorical choices.
  • Make sure you include more than one type of evidence, too. Let’s say you’re working on your essay and have gathered examples of alliteration to include as supporting evidence. That’s just one type of rhetorical choice, and it’s hard to make a credible argument if you’re only looking at one type of evidence. To fix that issue, reread the text again looking for patterns in word choice and syntax, meaningful figurative language and imagery, literary devices, and other rhetorical choices, looking for additional types of evidence to support your argument.
  • After you include evidence, offer your own interpretation and explain how this evidence proves the point you make in your thesis.
  • Don’t summarize or speak generally about the author and the text. Everything you write must be backed up with evidence.
  • Don’t let quotes speak for themselves. After every piece of evidence you include, make sure to explain your interpretation. Also, connect the evidence to your overarching argument.

Sophistication (0-1 point)

In this case, sophistication isn’t about how many fancy vocabulary words or how many semicolons you use. According to College Board , one point can be awarded to AP Lang Rhetorical Analysis essays that “demonstrate sophistication of thought and/or a complex understanding of the rhetorical situation” in any of these three ways:

  • Explaining the significance or relevance of the writer’s rhetorical choices.
  • Explaining the purpose or function of the passage’s complexities or tensions.
  • Employing a style that is consistently vivid and persuasive.

Note that you don’t have to achieve all three to earn your sophistication point. A good way to think of this rubric category is to consider it a bonus point that you can earn for going above and beyond in depth of analysis or by writing an especially persuasive, clear, and well-structured essay. In order to earn this point, you’ll need to first do a good job with your thesis, evidence, and commentary.

  • Focus on nailing an argumentative thesis and multiple types of evidence. Getting these fundamentals of your essay right will set you up for achieving depth of analysis.
  • Explain how each piece of evidence connects to your thesis.
  • Spend a minute outlining your essay before you begin to ensure your essay flows in a clear and cohesive way.
  • Steer clear of generalizations about the author or text.
  • Don’t include arguments you can’t prove with evidence from the text.
  • Avoid complex sentences and fancy vocabulary words unless you use them often. Long, clunky sentences with imprecisely used words are hard to follow.

AP Lang Rhetorical Analysis: Sample Prompt

The sample prompt below is published online by College Board and is a real example from the 2021 AP Exam. The prompt provides background context, essay instructions, and the text you need to analyze. For sake of space, we’ve included the text as an image you can click to read. After the prompt, we provide a sample high scoring essay and then explain why this AP Lang Rhetorical Analysis essay example works.

Suggested time—40 minutes.

(This question counts as one-third of the total essay section score.)

On February 27, 2013, while in office, former president Barack Obama delivered the following address dedicating the Rosa Parks statue in the National Statuary Hall of the United States Capitol building. Rosa Parks was an African American civil rights activist who was arrested in 1955 for refusing to give up her seat on a segregated bus in Montgomery, Alabama. Read the passage carefully. Write an essay that analyzes the rhetorical choices Obama makes to convey his message.

In your response you should do the following:

  • Respond to the prompt with a thesis that analyzes the writer’s rhetorical choices.
  • Select and use evidence to support your line of reasoning.
  • Explain how the evidence supports your line of reasoning.
  • Demonstrate an understanding of the rhetorical situation.
  • Use appropriate grammar and punctuation in communicating your argument.

AP Lang Rhetorical Analysis Essay Example

In his speech delivered in 2013 at the dedication of Rosa Park’s statue, President Barack Obama acknowledges everything that Parks’ activism made possible in the United States. Telling the story of Parks’ life and achievements, Obama highlights the fact that Parks was a regular person whose actions accomplished enormous change during the civil rights era. Through the use of diction that portrays Parks as quiet and demure, long lists that emphasize the extent of her impacts, and Biblical references, Obama suggests that all of us are capable of achieving greater good, just as Parks did.

Although it might be a surprising way to start to his dedication, Obama begins his speech by telling us who Parks was not: “Rosa Parks held no elected office. She possessed no fortune” he explains in lines 1-2. Later, when he tells the story of the bus driver who threatened to have Parks arrested when she refused to get off the bus, he explains that Parks “simply replied, ‘You may do that’” (lines 22-23). Right away, he establishes that Parks was a regular person who did not hold a seat of power. Her protest on the bus was not part of a larger plan, it was a simple response. By emphasizing that Parks was not powerful, wealthy, or loud spoken, he implies that Parks’ style of activism is an everyday practice that all of us can aspire to.

AP Lang Rhetorical Analysis Essay Example (Continued)

Even though Obama portrays Parks as a demure person whose protest came “simply” and naturally, he shows the importance of her activism through long lists of ripple effects. When Parks challenged her arrest, Obama explains, Martin Luther King, Jr. stood with her and “so did thousands of Montgomery, Alabama commuters” (lines 27-28). They began a boycott that included “teachers and laborers, clergy and domestics, through rain and cold and sweltering heat, day after day, week after week, month after month, walking miles if they had to…” (lines 28-31). In this section of the speech, Obama’s sentences grow longer and he uses lists to show that Parks’ small action impacted and inspired many others to fight for change. Further, listing out how many days, weeks, and months the boycott lasted shows how Parks’ single act of protest sparked a much longer push for change.

To further illustrate Parks’ impact, Obama incorporates Biblical references that emphasize the importance of “that single moment on the bus” (lines 57-58). In lines 33-35, Obama explains that Parks and the other protestors are “driven by a solemn determination to affirm their God-given dignity” and he also compares their victory to the fall the “ancient walls of Jericho” (line 43). By of including these Biblical references, Obama suggests that Parks’ action on the bus did more than correct personal or political wrongs; it also corrected moral and spiritual wrongs. Although Parks had no political power or fortune, she was able to restore a moral balance in our world.

Toward the end of the speech, Obama states that change happens “not mainly through the exploits of the famous and the powerful, but through the countless acts of often anonymous courage and kindness” (lines 78-81). Through carefully chosen diction that portrays her as a quiet, regular person and through lists and Biblical references that highlight the huge impacts of her action, Obama illustrates exactly this point. He wants us to see that, just like Parks, the small and meek can change the world for the better.

AP Lang Rhetorical Analysis Essay Example: Why It Works

We would give the AP Lang Rhetorical Analysis essay above a score of 6 out of 6 because it fully satisfies the essay’s 3 rubric categories: Thesis, Evidence and Commentary, and Sophistication . Let’s break down what this student did:

The thesis of this essay appears in the last line of the first paragraph:

“ Through the use of diction that portrays Parks as quiet and demure, long lists that emphasize the extent of her impacts, and Biblical references, Obama suggests that all of us are capable of achieving greater good, just as Parks did .”

This student’s thesis works because they make a clear argument about Obama’s rhetorical choices. They 1) list the rhetorical choices that will be analyzed in the rest of the essay (the italicized text above) and 2) include an argument someone else might disagree with (the bolded text above).

Evidence and Commentary:

This student includes substantial evidence and commentary. Things they do right, per the AP Lang Rhetorical Analysis rubric:

  • They include lots of specific evidence from the text in the form of quotes.
  • They incorporate 3 different types of evidence (diction, long lists, Biblical references).
  • After including evidence, they offer an interpretation of what the evidence means and explain how the evidence contributes to their overarching argument (aka their thesis).

Sophistication

This essay achieves sophistication according to the AP Lang Rhetorical Analysis essay rubric in a few key ways:

  • This student provides an introduction that flows naturally into the topic their essay will discuss. Before they get to their thesis, they tell us that Obama portrays Parks as a “regular person” setting up their main argument: Obama wants all regular people to aspire to do good in the world just as Rosa Parks did.
  • They organize evidence and commentary in a clear and cohesive way. Each body paragraph focuses on just one type of evidence.
  • They explain how their evidence is significant. In the final sentence of each body paragraph, they draw a connection back to the overarching argument presented in the thesis.
  • All their evidence supports the argument presented in their thesis. There is no extraneous evidence or misleading detail.
  • They consider nuances in the text. Rather than taking the text at face value, they consider what Obama’s rhetorical choices imply and offer their own unique interpretation of those implications.
  • In their final paragraph, they come full circle, reiterate their thesis, and explain what Obama’s rhetorical choices communicate to readers.
  • Their sentences are clear and easy to read. There are no grammar errors or misused words.

AP Lang Rhetorical Analysis Essay—More Resources

Looking for more tips to help your master your AP Lang Rhetorical Analysis Essay? Brush up on 20 Rhetorical Devices High School Students Should Know and read our Tips for Improving Reading Comprehension . If you’re ready to start studying for another part of the AP English Exam, find more expert tips in our How to Write the AP Lang Synthesis blog post.

Considering what other AP classes to take? Read up on the Hardest AP Classes .

  • High School Success

' src=

Christina Wood

Christina Wood holds a BA in Literature & Writing from UC San Diego, an MFA in Creative Writing from Washington University in St. Louis, and is currently a Doctoral Candidate in English at the University of Georgia, where she teaches creative writing and first-year composition courses. Christina has published fiction and nonfiction in numerous publications, including The Paris Review , McSweeney’s , Granta , Virginia Quarterly Review , The Sewanee Review , Mississippi Review , and Puerto del Sol , among others. Her story “The Astronaut” won the 2018 Shirley Jackson Award for short fiction and received a “Distinguished Stories” mention in the 2019 Best American Short Stories anthology.

  • 2-Year Colleges
  • Application Strategies
  • Best Colleges by Major
  • Best Colleges by State
  • Big Picture
  • Career & Personality Assessment
  • College Essay
  • College Search/Knowledge
  • College Success
  • Costs & Financial Aid
  • Data Visualizations
  • Dental School Admissions
  • Extracurricular Activities
  • Graduate School Admissions
  • High Schools
  • Law School Admissions
  • Medical School Admissions
  • Navigating the Admissions Process
  • Online Learning
  • Private High School Spotlight
  • Summer Program Spotlight
  • Summer Programs
  • Test Prep Provider Spotlight

College Transitions Sidebar Block Image

“Innovative and invaluable…use this book as your college lifeline.”

— Lynn O'Shaughnessy

Nationally Recognized College Expert

College Planning in Your Inbox

Join our information-packed monthly newsletter.

I am a... Student Student Parent Counselor Educator Other First Name Last Name Email Address Zip Code Area of Interest Business Computer Science Engineering Fine/Performing Arts Humanities Mathematics STEM Pre-Med Psychology Social Studies/Sciences Submit

What are your chances of acceptance?

Calculate for all schools, your chance of acceptance.

Duke University

Your chancing factors

Extracurriculars.

rhetorical analysis essay ap lang college board

How to Write the AP Lang Rhetorical Essay

Do you know how to improve your profile for college applications.

See how your profile ranks among thousands of other students using CollegeVine. Calculate your chances at your dream schools and learn what areas you need to improve right now — it only takes 3 minutes and it's 100% free.

Show me what areas I need to improve

What’s Covered:

What is the ap lang rhetorical essay, tips for writing the ap lang rhetorical essay.

  • AP Lang Rhetorical Essay Example

How Will AP Scores Affect College Chances?

The AP English Language Exam is one of the most common AP exams you can take. However, the average score on the exam in 2020 was a 2.96 out of 5. While this may seem a bit low, it is important to note that over 550,000 students take the exam annually. With some preparation and knowing how to study, it is totally possible to do well on this AP exam.

The AP Lang Rhetorical Essay is one section of the AP English Language Exam. The exam itself is 3 hours and 15 minutes long, and is broken into two sections. The first part of the exam is a 60 minute, 45-question multiple-choice section. The questions on this part of the exam will test your ability to read a passage and then interpret its meaning, style, and overall themes. After the multiple-choice section, there is a section lasting 2 hours and 15 minutes with three “free response” essays. This includes the synthesis essay, the rhetorical analysis essay, and the argument essay. 

  • In the synthesis essay , you will have to develop an argument using pieces of evidence provided to you. 
  • The argumentative essay will have you pick a side in a debate and argue for or against it.
  • The rhetorical essay requires that you discuss how an author’s written passage contributes to a greater meaning or theme. 

The rhetorical essay is perhaps the most unique of all AP Lang exam essays because it requires the test taker to analyze and interpret the deeper meanings of the passage and connect them to the author’s writing style and writing syntax in only 40 minutes. This essay can be the trickiest because it requires you to have knowledge of rhetorical strategies and then apply them to a passage you’ve never seen before.

1. Outline Your Essay Before Writing

One of the most important parts of the AP Lang essays is structuring your essay so that it makes sense to the reader. This is just as important as having good content. For this essay in particular, you’ll want to read the passage first and write a brief outline of your points before you begin the essay. This is because you will want to write the essay using the passage chronologically, which will be discussed in detail below.

2. Understand Rhetorical Strategies 

If you feel like you don’t know where to start as you prepare to study for the rhetorical essay portion of the exam, you aren’t alone. It is imperative that you have a grasp on what rhetorical strategies are and how you can use them in your essay. One definition of rhetoric is “language carefully chosen and arranged for maximum effect.” This can include types of figurative language (metaphor, simile, personification, pun, irony, etc.) elements of syntax (parallelism, juxtaposition, anthesis, anaphora, etc), logical fallacies, or persuasive appeals. Overall, there are many elements that you can analyze in an essay and having a good grasp on them through practice and memorization is important.

3. Keep the Essay Well Structured 

Even if you understand the various rhetorical strategies you can use, where do you begin? First of all, you’ll want to write a strong introduction that outlines the purpose of the piece. At the end of this introduction, you will write a thesis statement that encapsulates all the rhetorical strategies you discuss. Perhaps these are style elements, tone, or syntax. Be sure to be specific as you list these.

Next, you will create your body paragraphs. As you discuss the rhetorical elements in the piece and tie them back to the work’s meanings, be sure to discuss the points in chronological order. You don’t have to discuss every single strategy, but just pick the ones that are most important. Be sure to cite the line where you found the example. At the end of the essay, write a short conclusion that summarizes the major points above.

4. Be Sure to Explain Your Examples

As you write the essay, don’t just list out your examples and say something like “this is an example of ethos, logos, pathos.” Instead, analyze how the example shows that rhetoric device and how it helps the author further their argument. As you write the rhetorical essay, you’ll want to be as specific and detail-focused as possible. 

rhetorical analysis essay ap lang college board

Discover your chances at hundreds of schools

Our free chancing engine takes into account your history, background, test scores, and extracurricular activities to show you your real chances of admission—and how to improve them.

AP Lang Rhetorical Analysis Essay Example

Below is a prompt and example for a rhetorical essay, along with its score and what the writer did well and could have improved:

The passage below is an excerpt from “On the Want of Money,” an essay written by nineteenth-century author William Hazlitt. Read the passage carefully. Then write an essay in which you analyze the rhetorical strategies Hazlitt uses to develop his position about money.

rhetorical analysis essay ap lang college board

Student essay example:

In his essay, Hazlitt develops his position on money through careful use of adjectives and verbs, hypothetical situations, and images. His examples serve to impress upon the reader the highly negative consequences of being in “want of money.”

Hazlitt’s word choice in his opening phrase provides an example of his technique in the rest of the essay. It is not necessary to follow “literally” with “truly” yet his repetition of the same ideas emphasizes his point. In his next sentence, one that lasts forty-six lines, Hazlitt condignly repeats similar ideas, beating into his audience the necessity of having money in this world. The parallelism throughout that one long sentence, “it is not to be sent for to court, or asked out to dinner…it is not to have your own opinion consulted or sees rejected with contempt..” ties the many different situations Haziltt gives together. What could have become a tedious spiel instead becomes a melodious recitation, each example reminding you of one before it, either because of the similarities in structure or content. Hazlitt addresses many different negative effects of not having money but manages to tie them together with his rhetorical strategies. 

The diction of the passage fully relays Hazlitt’s position about money. In every example he gives a negative situation but in most emphasizes the terrible circumstance with strong negative adjectives or verbs. “Rejected,” “contempt,” “disparaged,” “scrutinized,” “irksome,” “deprived,” “assailed” “chagrin;” the endless repetition of such discouragement shows how empathetically Hazlitt believes money is a requisite for a happy life. Even the irony of the last sentences is negative, conveying the utter hopelessness of one without money. Through one may have none in life, pitiless men will proceed to mock one’s circumstances, “at a considerable expense” after death! 

In having as the body of his essay one long sentence, Hazlitt creates a flow that speeds the passage along, hardly giving the reader time to absorb one idea before another is thrown at him. The unceasing flow is synonymous with Hazlitt’s view of the life of a person without money: he will be “jostled” through life, unable to stop and appreciate the beauty around him or to take time for his own leisure. 

The score on this essay was a 6 out of 6. This essay started out very strong as the student had a concrete thesis statement explaining the strategies that Hazlitt used to develop his position on money as well as Hazlitt’s belief on the topic. In the thesis statement, the student points out that adjectives, verbs, hypothetical situations, and images help prove Hazlitt’s point that wanting money can be problematic. 

Next, the student broke down their points into three main subsections related to their thesis. More specifically, the student first discusses word choice of repetition and parallelism. When the student discusses these strategies, they list evidence in the paragraph that can be found chronologically in Hazlitt’s essay. The next paragraph is about diction, and the student used specific adjectives and verbs that support this idea. In the last paragraph, the student emphasized how the speed and flow of the essay helped describe Hazlitt’s viewpoint on life. This last concluding sentence is particularly thoughtful, as it goes beyond the explicit points made in the essay and discusses the style and tone of the writing. 

It is important to remember that in some ways, the rhetorical essay is also an argumentative essay, as the student must prove how certain rhetorical strategies are used and their significance in the essay. The student even discussed the irony of the paragraph, which is not explicit in the passage.

Overall, this student did an excellent job organizing and structuring the essay and did a nice job using evidence to prove their points. 

Now that you’ve learned about the AP Lang rhetorical essay, you may be wondering how your AP scores impact your chances of admission. In fact, your AP scores have relatively little impact on your admissions decision , and your course rigor has much more weight in the application process.

If you’d like to know your chances of admission, be sure to check out our chancing calculator! This tool takes into account your classes, extracurriculars, demographic information, and test scores to understand your chances at admission at over 600 schools. Best of all, it is completely free!

rhetorical analysis essay ap lang college board

Related CollegeVine Blog Posts

rhetorical analysis essay ap lang college board

logo-type-white

AP® English Language

3 ap® english language rhetorical essay strategies.

  • The Albert Team
  • Last Updated On: March 1, 2022

3_ap_english_language rhetorical essay strategies

The AP® English Language rhetorical essay can be nightmare inducing for some AP® students, but there is no need for fear. In this exam review we will lay out helpful strategies to get you through the rhetorical essays in no time.

Rhetorical Strategy #1: Dissecting the Prompt

The first rhetorical essay strategy is to dissect the prompt. Understanding what the rhetorical essay wants from you is essential. It is important for you to read the prompt carefully for every essay, but critical reading is even more essential to the rhetorical essay. Your rhetorical prompt that you will be given for the AP® English Language exam will contain two elements. The first element is the concrete task that the prompt is asking of you, which is always to analyze the passage that follows. The second part of the prompt is a more abstract task that is not directly asked for in the prompt, but it is implied. By completely understanding both parts of the prompt, you will be able to give a complete essay that will get you to a higher score.

One example of a prompt from an AP® English Language rhetorical essay is this one from the 2008 exam . The prompt reads:

“In the following passage from The Great Influenza, an account of the 1918 flu epidemic, author John M. Barry writes about scientists and their research. Read the passage carefully. Then, in a well-written essay, analyze how Barry uses rhetorical strategies.”

Here you can see the concrete task that the examiners are asking. They want you to analyze the passage for rhetorical strategies; however, you must figure out what you are analyzing the passage for. That is the more abstract concept that you need to dissect the prompt to find. In the case of Barry’s passage you will need to analyze how he uses rhetorical strategies in order to portray scientific research. We know this, because if you look at the prompt, it specifically states what Barry did in his work, which was to write about science and research. That is your abstract task.

Once you have found your concrete task and your abstract task, a great strategy is to write it down to keep you focused throughout your essay . Using the example above this would look like the following:

Analyze how Barry uses rhetorical strategies in order to portray scientific research.

That sentence is what you must follow when writing your essay, and if you successfully keep to this task, then you will move closer to that high score.

Rhetorical Essay Strategy #2: Stick to the Format

This next rhetorical essay strategy is the key to great organization and structure that will put your test anxiety to bed. There is a simple paragraph structure for the body paragraphs of the AP® English Language rhetorical essay that will allow you to think, write, and score higher, faster. You need to begin each body paragraph with an assertion or claim. That is the point that you are trying to make clear to your audience what you will be proving. A great example of this is from the 2006 AP® English Language rhetorical essay. Below is student 2B’s opening sentence for her first body paragraph.

“The diction of the passage fully relays Hazlitt’s position about money ( student 2B ).”

You can see how the student directly asserts what he or she will be proving in this statement. The next step in constructing your body paragraph is to give one to two pieces of textual evidence. Be sure to state why these quotations relate back to your claim, otherwise they will be deemed irrelevant by the examiners. An example of this is the next sentence in student 2B’s body paragraph about diction. Here, the student brings in elements from the text to support his or her claim about Hazlitt using diction.

“’Rejected’, ‘contempt’, ‘disparaged’, ‘scrutinized’, ‘irksome’, ‘deprived’, ‘assailed’, ‘chagrin’; the endless repetition of such discouragement shows just how emphatically Hazlitt money is requisite for happy life (student 2B).”

The final part of this strategy for conquering the body paragraphs of your rhetorical essays is to end those body paragraphs with a thorough analysis. This is the aspect of the exam where you can put your way of looking at the text into your essay.

An example of this is at the end of student 2B’s body paragraph where he or she states, “The irony of the last sentences is negative, conveying the utter hopelessness of one without money. Though one may have none in life, pitiless men will continue to mock one’s circumstances even after death! (student 2B)”

This analysis of the text adds to the textual examples above and continues to bring in new logic from the student.

When this format of a body paragraph is followed, then it is extremely effective. The essay becomes clear, assertive, and easy to follow for the examiners. Follow this rhetorical essay strategy and you are even closer to getting that 5 on the exam.

Rhetorical Essay Strategy #3: LORA

As you are looking at your AP® English Language rhetorical essay prompt and passage it is important to remember the mnemonic device, LORA. LORA stands for Language, Organization, and Rhetorical Appeals. These elements will help you form your argument.

When you read through your passage you want to think about how the author is utilizing language. Is he or she using figurative language effectively? Is there imagery within the passage? Does the diction of the passage make it more rhetorically persuasive? You should not use all of these, but picking one and analyzing it clearly in one paragraph will keep you focused on how the author uses rhetoric, which is the main task of this essay.

An example of this was in the 2006 AP® English Language rhetorical essay. Student 2A begins his or her first body paragraph with, “One of Hazlitt’s most effective methods of promoting the importance of money is his strong diction (student 2A).” This student begins his or her essay with focusing on diction as how the language is used. He or she then goes on to explain why diction betters Hazlitt’s argument, which is exactly what you must do for your own rhetorical essay.

The organization of the author is the next part of your answer to the prompt. You want to look at how the author organized his or her ideas within the passage to support his or her own argument. By pointing out the organization, or structure, of the work and how it adds to the overall persuasiveness, you will bring two of the three most important elements of rhetoric together in your essay.

After organization you need to look at the rhetoric appeals. You may know them by the names logos, pathos, and ethos. It is suggested that you cover as many of these as possible; however, if time does not permit or if the passage uses one more than the other, then you should focus on one appeal.

One example of using pathos in an essay is from student 2A from the 2006 prompt. “Hazlitt plays on the audience’s heartstrings for more than enough time to convince them of the importance of having money (student 2A).” While it would have been better for the student to directly say that this is pathos, he or she does thoroughly explain the appeal to the passions, or pathos.

Key Takeaways

When taking the AP® English Language rhetoric essay you just need to remember these three rhetorical essay strategies: dissect the prompt, follow the format, and always include LORA. If you can follow them, then you are already on your way to a 5 on the AP® English Language exam .

Let’s put everything into practice. Try this AP® English Language practice question:

Rhetorical Considerations AP® English Language Practice Question

Looking for more AP® English Language practice?

Check out our other articles on AP® English Language .

You can also find thousands of practice questions on Albert.io. Albert.io lets you customize your learning experience to target practice where you need the most help. We’ll give you challenging practice questions to help you achieve mastery of AP® English Language.

Start practicing here .

Are you a teacher or administrator interested in boosting AP® English Language student outcomes?

Learn more about our school licenses here .

Interested in a school license?​

Popular posts.

AP® Physics I score calculator

AP® Score Calculators

Simulate how different MCQ and FRQ scores translate into AP® scores

rhetorical analysis essay ap lang college board

AP® Review Guides

The ultimate review guides for AP® subjects to help you plan and structure your prep.

rhetorical analysis essay ap lang college board

Core Subject Review Guides

Review the most important topics in Physics and Algebra 1 .

rhetorical analysis essay ap lang college board

SAT® Score Calculator

See how scores on each section impacts your overall SAT® score

rhetorical analysis essay ap lang college board

ACT® Score Calculator

See how scores on each section impacts your overall ACT® score

rhetorical analysis essay ap lang college board

Grammar Review Hub

Comprehensive review of grammar skills

rhetorical analysis essay ap lang college board

AP® Posters

Download updated posters summarizing the main topics and structure for each AP® exam.

Calculate for all schools

Your chance of acceptance, your chancing factors, extracurriculars, ap lang analysis essay.

I'm struggling with understanding the AP Lang analysis essay format and process. Can someone break down the steps for me or share resources that will help me tackle these essays?

Sure, I'd be happy to break down the steps for you. The AP Lang analysis essay, also known as the rhetorical analysis essay, requires you to analyze a text (usually a speech or an article) and explain how the author uses various rhetorical strategies to achieve their purpose.

Here are the steps you should follow:

1. Read the prompt: Carefully read the given prompt, which will provide you with the context and specific instructions for the essay.

2. Analyze the text: Read the provided text at least twice. The first reading should be a quick one, to get the overall gist. The second reading should be more focused, noting specific rhetorical devices, strategies, and examples of their use.

3. Take notes: As you read and analyze the text, jot down the main points, rhetorical strategies used, and specific evidence that support your analysis.

4. Create a thesis statement: Based on your understanding of the text and prompt, come up with a clear and concise thesis statement that outlines your main argument.

5. Outline your essay: Create a brief outline for your essay, which will include an introduction, body paragraphs (each focusing on one or two rhetorical strategies), and a conclusion.

6. Write the introduction: Begin with an engaging opening that provides context for the text and its author, and then introduce your thesis statement.

7. Write body paragraphs: For each body paragraph, start with a topic sentence that clearly outlines the rhetorical strategy you will analyze. Then, provide specific examples from the text to support your analysis, explaining how the author uses the rhetorical devices to achieve their purpose. Make sure you're focusing on analysis, rather than mere summarization of the text.

8. Write the conclusion: In your conclusion, restate your thesis statement and summarize the key points of your analysis. You can also offer implications or broader significance if relevant.

9. Proofread and revise: Carefully review your essay for clarity, grammar, and spelling errors, and make any necessary revisions.

For additional resources, consider checking out the College Board's AP Lang resources ( https://apcentral.collegeboard.org/courses/ap-english-language-and-composition) and CollegeVine ( https://blog.collegevine.com/how-to-write-the-ap-lang-rhetorical-essay/). Also, practice writing these essays under time constraints to get familiar with the process and improve your skills.

Remember, practice is crucial for mastering the AP Lang analysis essay format. Keep practicing, and you'll become more confident and adept at tackling these essays. Good luck!

About CollegeVine’s Expert FAQ

CollegeVine’s Q&A seeks to offer informed perspectives on commonly asked admissions questions. Every answer is refined and validated by our team of admissions experts to ensure it resonates with trusted knowledge in the field.

pep

Find what you need to study

Rhetorical Devices List w/ Examples

37 min read • may 10, 2022

Sumi Vora

List of Rhetorical Devices & Terms

Taking AP English Language? This is a list of main rhetorical device terms that you should know for the exam as well as definitions & examples for each. These terms will mostly show up on the multiple-choice section, so it’s important to be able to identify them in a work of writing, but you won’t actually have to use the device in your own writing. Each term includes a definition, an example of the rhetorical device being used in a text, and an example of analysis that might be used in an essay.

In your essays, you will need to identify which devices are used and their effect on the work as a whole. Sometimes, a writer will use a device (for example: alliteration), but it doesn’t have a huge effect on the work or the writer’s argument. In that case, don't spend an entire paragraph talking about alliteration. You need to focus on what matters most, and you need to specifically show how  these choices make the work effective, and why they are so important. Yes, the rhetorical analysis essay is an argument essay just like the other two.

You aren't required to use rhetorical vocabulary in your essays at all — in fact, it’s probably better if you don’t. If you force the vocabulary into your essay, you risk sounding clunky, and the vocabulary almost always leads you to switch to passive voice. Instead, just describe what is happening! (ex: The author uses imagery → The author’s vivid images). This method also ensures that you are showing how the device is contributing to the work, rather than simply identifying it.

And, without further ado… Here are some rhetorical devices you should know for the AP Lang exam:

1. aesthetic

Definition: This rhetorical device references to artistic elements or expressions within a textual work

Example of aesthetic: 

“The Flapper” by Dorothy Parker (1922)

The Playful flapper here we see,

The fairest of the fair.

She's not what Grandma used to be, —

You might say, au contraire.Her girlish ways may make a stir,

Her manners cause a scene,

But there is no more harm in her

Than in a submarine.

She nightly knocks for many a goal

The usual dancing men.

Her speed is great, but her control

Is something else again.

All spotlights focus on her pranks.

All tongues her prowess herald.

For which she well may render thanks

To God and Scott Fitzgerald.

Her golden rule is plain enough —

Just get them young and treat them

Analysis:  Parker describes the aesthetic  of flapper culture in her poem in order to support women who defied social norms and who adopted more liberal attitudes towards makeup, drinking, smoking, and sex.

Note: aesthetic is not necessarily a specific device; it is the bigger picture. An author would use a rhetorical device (e.g. imagery, allusions, etc.) to achieve a certain aesthetic.

2. allegory

Definition: This rhetorical device references the expression by means of symbolic fictional figures and actions of truths or generalizations about human existence

Allegory Example:  

Animal Farm  by George Orwell (1945)

All that year the animals worked like slaves. But they were happy in their work; they grudged no effort or sacrifice, well aware that everything they did was for the benefit of themselves and those of their kind who would come after them, and not for a pack of idle, thieving human beings.

Analysis:  In George Orwell’s allegorical  novel Animal Farm , overworked farm animals rise up against their owner and subscribe to the concepts of Animalism, which proclaims that “all men are enemies” and “all animals are comrades.” The animals, who now work “like slaves” for the “benefit of themselves and those of that their kind,” run a society that mirrors that of the Russian Revolution. Orwell’s use of animals to describe contemporary political events creates distance between his novel and his potentially incendiary critique of the rise of Communism, which makes the topic more approachable.

3. alliteration  

Definition: This rhetorical device references the repetition of the same sound at the beginning of successive words

Alliteration Example:  

Ronald Reagan’s Address at the Vietnam Veteran’s Memorial (1988)

Our liberties, our values — all for which America stands — is safe today because brave men and women have been ready to face the fire at freedom's front. And we thank God for them.

Analysis:  Reagan acknowledges that the veterans of the Vietnam War were prepared to “face the fire at freedom’s front.” Through his use of alliteration , Reagan emphasizes the soldiers’ willingness to sacrifice themselves for freedom, focusing the audience’s attention on the value of the veterans’ deeds.

4. allusion

Definition: This rhetorical device is a reference, explicit or implicit, to something in previous literature or history

Allusion Example:  

“I Have a Dream” by Martin Luther King, Jr. (1963)

Five score years ago, a great American, in whose symbolic shadow we stand today, signed the Emancipation Proclamation. This momentous decree came as a great beacon light of hope to millions of Negro slaves who had been seared in the flames of withering injustice. It came as a joyous daybreak to end the long night of their captivity.

Analysis:  King begins his speech with both an indirect and direct allusion  to Abraham Lincoln’s “Emancipation Proclamation.” The first phrase of King’s speech, “Five score years ago,” directly mirrors Lincoln’s historic speech, which opens with “four score and seven years ago.” By associating himself with a prominent figure in the fight against injustice, King implies that he shares Lincoln’s values and establishes a sympathetic relationship with his audience.

5. ambiguity

Definition: This rhetorical device references a word, phrase, or sentence whose meaning can be interpreted in more than one way

Ambiguity Example:  

The Awakening  by Kate Chopin (1899)

Exhaustion was pressing upon and overpowering her.

"Good-by— because I love you." He did not know; he did not understand. He would never understand. Perhaps Doctor Mandelet would have understood if she had seen him — but it was too late; the shore was far behind her. And her strength was gone.

Analysis:  At the end of Kate Chopin’s The Awakening , Edna lends herself to the tide with the vague last words, “good-by— because I love you,” leaving Victor to question whether her death was intentional. Chopin’s use of ambiguity  to depict Enda’s death illustrates Victor’s lack of closure and his feeling of utter helplessness and confusion as he watches his loved one, both physically and metaphorically, swept away by the current.

Definition: This rhetorical device references an extended comparison between two things/instances/people etc. that share some similarity to make a point

Analogy Example:  

“What True Education Should Do” by Sydney J. Harris (1994)

Pupils are more like oysters than sausages. The job of teaching is not to stuff them and then seal them up, but to help them open and reveal the riches within. There are pearls in each of us, if only we knew how to cultivate them with ardor and persistence.

Analysis:  Harris compares students to oysters whom we should help “open and reveal the riches within.” Through her analogy , Harris establishes a basis on which readers can shift their perspective. Rather than simply listing specific traits of students, Harris helps her readers change their perception of how students should be treated, and gives readers a concrete and memorable lense through which readers should view the classroom.

7. anaphora

Definition: This rhetorical device references repetition of the same word or group of words at the beginning of successive clauses, sentences, or lines

Anaphora Example:  

I have a dream that one day even the state of Mississippi, a state sweltering with the heat of injustice, sweltering with the heat of oppression, will be transformed into an oasis of freedom and justice. I have a dream that my four little children will one day live in a nation where they will not be judged by the color of their skin but by the content of their character.

I have a dream today.

Analysis:  King repeats the phrase, “I have a dream” to emphasize his vision for racial equality in the United States. By employing anaphora  to underscore his beliefs, King connects his ideas with a common motif, helping his audience follow his speech and make it more memorable. King thus invites his audience to share in his “dream,” as he reminds them that it is their dreams for a more equal future that unite their movement.

8. anecdote

Definition: This rhetorical device references a usually short narrative of an interesting, amusing, or biographical incident

Anecdote Example:  

“Gender Equality is Your Issue Too” by Emma Watson (2014)

I started questioning gender-based assumptions when at eight I was confused at being called “bossy,” because I wanted to direct the plays we would put on for our parents—but the boys were not. When at 14 I started being sexualized by certain elements of the press. When at 15 my girlfriends started dropping out of their sports teams because they didn’t want to appear “muscly.” When at 18 my male friends were unable to express their feelings. I decided I was a feminist and this seemed uncomplicated to me.

Analysis:  By sharing a short anecdote  about being “sexualized” and called “bossy,” while acknowledging her male friends being “unable to express their feelings,” Watson establishes her authority to speak on gender-related issues, and she appeals to her audience’s sense of emotion and empathy as she aims to establish a common experience between both men and women in the United Nations.

9. antithesis

Definition: This rhetorical device references the rhetorical contrast of ideas by means of parallel arrangements of words, clauses, or sentences

Antithesis Example:  

Neil Armstrong’s moon landing (1969)

“That’s one small step for man; one giant leap for mankind”

Analysis:  Armstrong’s antithesis  serves to highlight the monumental impact that the moon landing will have on the human race. By contrasting his “small step” with the “giant” effect that this step will have, he emphasizes its significance.

10. assonance

Definition: the repetition of vowel sounds but not consonant sounds

Assonance Example:  

The Color Purple  by Alice Walker (1982)

She got sicker an sicker.

Finally, she ast Where it is?

I say God took it.

He took it. He took it while I was sleeping. Kilt it out there in the woods. Kill this one too, if he can.

Analysis:  In her second letter to God, Celie describes her mother getting “sicker an sicker” and the way God “kilt” her first child in the woods. The repetition of the “i” sound creates a staccato and rhythmic quality to the letter while still creating a thin, ill-sounding intonation.

Note: assonance is often associated with euphony : soothing and pleasant sounds.

11. asyndeton

Definition: conjunctions are omitted, producing a fast-paced and rapid prose

Asyndeton Example:  

“Duty, Honor, Country” by General Douglas MacArthur (1962)

Duty, Honor, Country: Those three hallowed words reverently dictate what you ought to be, what you can be, what you will be. They are your rallying points: to build courage when courage seems to fail; to regain faith when there seems to be little cause for faith; to create hope when hope becomes forlorn.

Analysis:  In his speech, MacArthur rallies the United States army with three simple words: “duty, honor, country.” MacArthur’s asyndeton  creates a powerful and concise phrase that galvanizes his men through its simplicity. Because the conjunctions have been omitted, MacArthur’s phrase reads like a chant in which each word is emphasized equally. This rhythmic phrase is thus very easy to remember and to repeat, which allows MacArthur to invigorate and prepare his army.

12. chiasmus

Definition: repetition of ideas in inverted order

Example:  John F. Kennedy’s Inaugural Address (1971)

The energy, the faith, the devotion which we bring to this endeavor will light our country and all who serve it — and the glow from that fire can truly light the world. And so, my fellow Americans: ask not what your country can do for you — ask what you can do for your country.

Analysis:  In his 1971 Inaugural Address, Kennedy encourages his audience to have faith in their generation and in their country in the midst of a trying Cold War. Kennedy attempts to unite the audience under a national identity and purpose, inviting them to consider not what their “country can do for” them, but what they “can do for” their country. By employing chiasmus , Kennedy highlights the difference between an archaic mentality and the attitude that he wants the country to adopt moving forward. Because Kennedy repeats the same simple ideas, he also creates a memorable phrase that allows his message to spread easily among the American people.

13. colloquial

Definition: characteristic of spoken or written communication that seeks to imitate informal speech

Example:  Barack Obama’s message about political ‘wokeness’ (2019)

This idea of purity and you’re never compromised and you’re always politically woke and all that stuff; you should get over that quickly. The world is messy. There are ambiguities.

Analysis:  In his commentary regarding the call-out culture on the current socio-political stage, Obama uses the term “woke” to describe those who believe they are more aware of social injustices. By adopting a colloquial  expression, Obama molds his message to resonate with young Americans. Obama is thus able to connect with his audience by mimicking their language.

14. connotation

Definition: the set of associations implied by a word in addition to its literal meaning

Example:  “Black Men in Public Space” by Brent Staples (1986)

My first victim was a white woman, well dressed, probably in her early twenties. I came upon her late one evening on a deserted street in Hyde Park, a relatively affluent neighborhood in an otherwise mean, impoverished section of Chicago. As I swung onto the avenue behind her, there seemed to be a discreet, noninflammatory distance between us. Not so. She cast back a worried glance. To her, the youngish black man – a broad six feet two inches with a beard and billowing hair, both hands shoved into the pockets of a bulky military jacket – seemed menacingly close.

Analysis:  In his essay “Black Men in Public Space,” Brent Staples refers to the woman who runs away from him as his “victim” to whom he is “menacingly close,” which connotes violence and criminal activity. However, the actions that ensue do not match such connotations ; rather than attacking the woman, Staples simply walks down the avenue. By breaking the audience’s expectations, Staples highlights the misleading dialogue surrounding African-American men and forces his readers to confront their own racial biases.

Note: connotation and tone are very closely related. Often, an author will use words that carry certain connotations to establish a tone. You can use this idea in your essays to demonstrate tone by citing the connotative words the author uses to establish such a tone.

15. consonance

Definition: the repetition of consonant sounds, but not vowels, as in assonance

Example: “Jabberwocky” by Lewis Caroll (1871)

“Beware the Jabberwock, my son!

The jaws that bite, the claws that catch!

Beware the Jubjub bird, and shun

The frumious Bandersnatch!”

Analysis:  In Lewis Carol’s poem “Jabberwocky,” he warns against the Jabberwock’s “jaws” and the “Jubjub bird,” repeating the “j” sound. Carol uses consonance  to create dissonant and almost disorienting sounds through harsh, hard tones, which emphasize the obnoxious nature of the Jabberwocky. Because of the abundance of consonants, the poem reads similar to a tongue-twister, which further serves to disorient the reader and make them feel as if they are in a completely different world.

Note: consonance can be associated with cacophony, or harsh, discordant sounds, if it uses “explosive consonants” such as B, C, CH, D, G, J, K, P, Q, T, X.

16. deductive reasoning

Definition: reasoning that works from the more general to the more specific, beginning with a theory that becomes a hypothesis, and using observations to confirm the original theory (top-down approach)

Example:  Mahatma Gandhi’s letter to British Viceroy Lord Irwin (1930)

If I have equal love for your people with mine, it will not long remain hidden. It will be acknowledged by them, even as the members of my family acknowledged after they had tried me for several years. If the people join me, as I expect they will, the sufferings they will undergo, unless the British nation sooner retraces its steps, will be enough to melt the stoniest hearts. The plan through civil disobedience will be to combat such evils as I have sampled out. If we want to sever the British connection it is because of such evils. When they are removed, the path becomes easy. Then the way to friendly negotiation will be open. If the British commerce with India is purified of greed, you will have no difficulty in recognizing our independence.

Analysis:  In his letter to Lord Irwin, Gandhi uses a series of if-then statements to defend India’s call for independence through civil disobedience. Gandhi begins by establishing his “equal love” for the British people and mentioning that if they join him in his protests, it will “melt the stoniest of hearts” in the British government, forcing the British to “retrace their steps” and remove the “evils” in the current British regime. If the evils are removed, Gandhi promises, the “way to friendly negotiation will be open.” By articulating his position with deductive reasoning , Gandhi appeals to Lord Irwin’s logic and maintains that the Indian people are not acting irrationally. Gandhi provides Lord Irwin with only one logical option: purify the British commerce system of greed and open the table to negotiate with India.

17. denotation

Definition: the literal meaning of a word, the dictionary definition

Example:  “Gender Equality is Your Issue Too” by Emma Watson (2014)

I was appointed six months ago and the more I have spoken about feminism the more I have realized that fighting for women’s rights has too often become synonymous with man-hating. If there is one thing I know for certain, it is that this has to stop.

For the record, feminism by definition is: “The belief that men and women should have equal rights and opportunities. It is the theory of the political, economic and social equality of the sexes.

Analysis:  By explicitly defining feminism as “the belief that men and women should have equal rights and opportunities,” Watson juxtaposes the denotation  of feminism with the connotations with which it is associated. Watson directly confronts the misconceptions regarding feminism to quell any opposition regarding such misconceptions, and she appeals to a credible source — the dictionary — to support her claims and establish her own authority over the matter.

Note: denotation is almost always used in contrast with connotation. Authors will often define a word to clarify its meaning, which suggests that the connotations of the term do not match how the author wants the audience to view that term.

18. diction

Definition: a writer's choice of words, phrases, sentence structures, and figurative language, which combine to help create meaning

Example:  “On Dumpster Diving” by Lars Eighner (1992)

Canned goods are among the safest foods to be found in Dumpsters but are not utterly foolproof. Although very rare with modern canning methods, botulism is a possibility. Most other forms of food poisoning seldom do lasting harm to a healthy person, but botulism is almost certainly fatal and often the first symptom is death. Except for carbonated beverages, all canned goods should contain a slight vacuum and suck air when first punctured. Bulging, rusty, and dented cans and cans that spew when punctured should be avoided, especially when the contents are not very

acidic or syrupy.

Analysis:  Eighner employs empirical diction  to describe the process of dumpster diving, which is generally considered a dishonorable and crude practice. Eighner details the “fatal” effects of “botulism,” and provides a practical assessment of “modern canning methods,” instructing readers to avoid “bulging, rusty, and dented cans” and to look for a “slight vacuum” in canned goods. By analyzing the process of dumpster diving through a scientific lens, Eighner emphasizes that those who dumpster dive are not inferior to their store going counterparts, and he suggests that dumpster diving can be a practical hobby for anyone, even if it is not done out of necessity.

19. didactic

Definition: tone; instructional, designed to teach an ethical, moral, or religious lesson

Example:  “Advice to Youth” by Mark Twain (1882)

First, then. I will say to you my young friends — and I say it beseechingly, urgently — Always obey your parents, when they are present. This is the best policy in the long run because if you don’t, they will make you. Most parents think they know better than you do, and you can generally make more by humoring that superstition than you can by acting on your own better judgment.

Analysis:  In his satire “Advice to Youth,” Twain adopts a didactic  tone that mimics that of many parents chastising their children. He instructs youth to “always obey [their] parents” because “most parents think they know better than” their children. By using a familiar instructional tone while mocking parental attitude, Twain appeals to his credibility by establishing that he too has faced criticism from his parents. By recognizing a common experience, Twain builds a rapport with his young audience, making them more receptive to his message.

Note: Generally, essays with a very didactic tone are ineffective, so they don’t have much rhetorical merit. Twain’s speech is instead a satire of the didactic tone many parents adopt, which allows him to connect with his audience in their mutual scorn for some parents’ sanctimonious attitude.

20. elegiac

Definition: a tone involving mourning or expressing sorrow for that which is irrecoverably past

Example:  Ronald Reagan’s address following the explosion of the Challenger Space Shuttle (1986)

Today is a day for mourning and remembering. Nancy and I are pained to the core by the tragedy of the shuttle Challenger. We know we share this pain with all of the people of our country. This is truly a national loss. For the families of the seven, we cannot bear, as you do, the full impact of this tragedy. But we feel the loss, and we're thinking about you so very much.

Analysis:  At the beginning of his address, Reagan adopts an elegiac  tone, declaring that “today is a day for mourning and remembering.” He describes the deaths of the astronauts as a “national loss” that pains “all of the people” in the United States. By taking the time to recognize the tragic loss of the astronauts and by empathizing with the American people’s shock at the explosion, Reagan appeals to his audience’s grief and establishes an emotional connection with them before he begins speaking about the future of the United States space exploration program.

21. epistrophe

Definition: ending a series of lines, phrases, clauses, or sentences with the same word or words

Example:  Madelynn Albright’s commencement speech for Mount Holyoke College (1997)

As you go along your own road in life, you will, if you aim high enough, also meet resistance, for as Robert Kennedy once said, “if there’s nobody in your way, it’s because you’re not going anywhere.” But no matter how tough the opposition may seem, have courage still—and persevere.

There is no doubt, if you aim high enough, that you will be confronted by those who say that your efforts to change the world or improve the lot of those around you do not mean much in the grand scheme of things. But no matter how impotent you may sometimes feel, have courage still — and persevere.

It is certain, if you aim high enough, that you will find your strongest beliefs ridiculed and challenged; principles that you cherish may be derisively dismissed by those claiming to be more practical or realistic than you. But no matter how weary you may become in persuading others to see the value in what you value, have courage still—and persevere.

Inevitably, if you aim high enough, you will be buffeted by demands of family, friends, and employment that will conspire to distract you from your course. But no matter how difficult it may be to meet the commitments you have made, have courage still—and persevere.

Analysis:  In her commencement speech, Albright encourages women to stand firm and to “aim high,” despite the prevalence of gender inequality. Albright recognizes that women face opposition and glass ceilings, but she urges them to “have courage still— and persevere,” repeating the phrase after each challenge she discusses. Like her attitude towards success, Albright’s speech always returns to the idea that women must “have courage still — and persevere,” regardless of the obstacles presented to her. Albright’s motto to “have courage still—and persevere” is the most prominent part of her speech, and remains consistent even when the rest of her speech shifts, which mirrors the outlook that Albright endorses.

Definition: appealing to credibility

Example:  “Letter from a Birmingham Jail” by Martin Luther King, Jr. (1963)

I think I should indicate why I am here in Birmingham since you have been influenced by the view which argues against "outsiders coming in." I have the honor of serving as president of the Southern Christian Leadership Conference, an organization operating in every southern state, with headquarters in Atlanta, Georgia. We have some eighty-five affiliated organizations across the South, and one of them is the Alabama Christian Movement for Human Rights. Frequently we share staff, educational and financial resources with our affiliates. Several months ago the affiliate here in Birmingham asked us to be on call to engage in a nonviolent direct action program if such were deemed necessary. We readily consented, and when the hour came we lived up to our promise. So I, along with several members of my staff, am here because I was invited here. I am here because I have organizational ties here.

Analysis:  King mentions that he is the “president of the Southern Christian Leadership Conference” that operates in “every southern state” and has “eighty-five affiliated organizations across the South.” He also emphasizes that he is in Birmingham because he was “invited” due to “organizational ties.” King spends a significant amount of time describing his credentials and his affiliation with the Church, which not only creates a common experience among the clergymen and himself but also establishes King as a respectable man with significant accomplishments. Because many white southerners believed that African Americans were inferior to themselves, King takes the time to appeal to his own credibility and authority in hopes that the clergymen will view him as their equal and will respect his message.

Note: please don’t write “appeals to ethos/pathos/logos.” Instead, try “appeals to credibility/emotion/logic,” or go further to describe specifically which emotion or credentials the author appeals to.

23. extended metaphor

Definition: differs from a regular metaphor in that several comparisons similar in theme are being made

Example:  “Is Google Making Us Stupid?” by Nicholas Carr (2008)

Over the past few years I’ve had an uncomfortable sense that someone, or something, has been tinkering with my brain, remapping the neural circuitry, reprogramming the memory. My mind isn’t going—so far as I can tell—but it’s changing. I’m not thinking the way I used to think. I can feel it most strongly when I’m reading. Immersing myself in a book or a lengthy article used to be easy. My mind would get caught up in the narrative or the turns of the argument, and I’d spend hours strolling through long stretches of prose.

Analysis:  Carr employs an extended metaphor  to liken his brain to a machine, suggesting that something “has been tinkering” with his brain, “remapping” and “reprogramming” his “neural circuitry.” By comparing his brain to a machine, Carr conveys his feeling that he is a slave to his computer and his sense of disconnectedness from his brain. Rather than being in harmony with his mind, he describes his brain as a separate entity. Carr’s metaphor also highlights the increasing influence of technology in modern life — so much so that our brains themselves have become computers.

24. imagery

Definition: descriptive language that provides vivid images that evoke the senses

Example:   Last Child in the Woods  by Richard Louv (2008)

In our useful boredom, we used our fingers to draw pictures on fogged glass as we watched telephone poles tick by. We saw birds on the wires and combines in the fields. We were fascinated with roadkill, and we counted cows and horses and coyotes and shaving-cream signs. We stared with a kind of reverence at the horizon, as thunderheads and dancing rain moved with us. We held our little plastic cars against the glass and pretended that they, too, were racing toward some unknown destination. We considered the past and dreamed of the future, and watched it all go by in the blink of an eye.

Analysis:  Louv recounts his experience staring out of the car window as a child with vivid imagery , describing watching “telephone poles tick by,” “birds on the wires,” “cows and horses and coyotes,” and “shaving-cream signs.” Louv jots seemingly disconnected images in short snippets, mimicking a car whizzing past an ever-changing landscape. The sharp images appeal to the reader’s sense of nostalgia as Louv allows them to witness their own youth “go by in the blink of an eye.”

25. inductive reasoning

Definition: reasoning that moves from specific observations to broader generalizations and theories; uses observations to detect patterns and regularities, and develops a hypothesis and later broader theories based on these observations (bottom-up approach)

Example:  “On Being a Cripple” by Nancy Mairs (1986)

"Cripple" seems to me a clean word, straightforward and precise. As a lover of words, I like the accuracy with which it describes my condition: I have lost the full use of my limbs. "Disabled," by contrast, suggests any incapacity, physical or mental. And I certainly don't like "handicapped," which implies that I have deliberately been put at a disadvantage, by whom I can't imagine (my God is not a Handicapper General), in order to equalize chances in the great race of life. These words seem to me to be moving away from my condition, to be widening the gap between word and reality. Most remote is the recently coined euphemism "differently-abled," which partakes of the same semantic hopefulness that transformed countries from "undeveloped" to "underdeveloped," then to "less developed," and finally to "developing" nations. People have continued to starve in those countries during the shift. Some realities do not obey the dictates of language.

Analysis:  Mairs begins by outlining her views on the word “cripple,” which “describes [her] condition” in a “straightforward and precise manner,” unlike vague terms such as “handicapped” and “differently-abled,” which widen “the gap between word and reality.” Much like “people have continued to starve” in underdeveloped nations despite the shift in nomenclature, Mairs scorns the “semantic hopefulness” that has led people to use less precise words to describe her condition, even though the disability itself cannot change. Mairs uses inductive reasoning  to conclude that “some realities do not obey the dictates of language” as she appeals to readers’ logic to deduce that using euphemisms to describe unfavorable circumstances is irrational and only serves to dilute the rectitude of precise language.

Definition: stating the opposite of what is said or meant

I hope you will treasure up the instructions which I have given you, and make them a guide to your feet and a light to your understanding. Build your character thoughtfully and painstakingly upon these precepts, and by and by, when you have got it built, you will be surprised and gratified to see how nicely and sharply it resembles everybody else’s.

Analysis:  Twain instructs youth to “treasure” his instructions and to construct their “character thoughtfully and painstakingly upon” the precepts they have read. However, Twain mentions that if they do so, they will be “surprised and gratified to see how nicely and sharply it resembles everybody else’s.” Twain’s irony  warns youth that if they simply obey their parents, they will not become a unique individual, and the unexpected ending to his satire reinforces his position that one should not mold themselves to meet societal norms.

27. juxtaposition

Definition: placing two or more things side by side for comparison or contrast

Example:   Silent Spring  by Rachel Carson (1962)

Along the roads, laurel, viburnum, and alder, great ferns and wildflowers delighted the traveler’s eye through much of the year. Even in winter, the roadsides were places of beauty, where countless birds came to feed on the berries and on the seed heads of the dried weeds rising above the snow. The countryside was, in fact, famous for the abundance and variety of its birdlife, and when the flood of migrants was pouring through in spring and fall people traveled from great distances to observe them. Others came to fish the streams, which flowed clear and cold out of the hills and contained shady pools where trout lay. So it had been from the days many years ago when the first settlers raised their houses, sank their wells, and built their barns.

Then a strange blight crept over the area and everything began to change. Some evil spell had settled on the community: mysterious maladies swept the flocks of chickens; the cattle and sheep sickened and died. Everywhere was a shadow of death. The farmers spoke of much illness among their families. In the town the doctors had become more and more puzzled by new kinds of sickness appearing among their patients. There had been several sudden and unexplained deaths, not only among adults but even among children, who would be stricken suddenly while at play and die within a few hours.

Analysis:  In her novel Silent Spring , Rachel Carson describes the beautiful American town with the cold, vapid town that it is destined to become due to climate change. She juxtaposes  the town’s “great ferns and wildflowers,” “birdlife,” and “clear and cold” streams with the “strange blight” that cast an “evil spell” on the community and the animals who have “sickened and died” from “mysterious maladies.” By creating such a sharp contrast between the present and the future, Carson coveys the magnitude of the climate crisis and emphasizes the urgency with which we must address it. Carson’s starkly contrasting images aim to evoke a strong emotional response in the reader that appeals to their sense of responsibility and citizenship.

Definition: appealing to logic

Example:  Greta Thunberg’s speech at the National Assembly in Paris (2019)

A lot of people, a lot of politicians, business leaders, journalists say they don't agree with what we are saying. They say we children are exaggerating, that we are alarmists. To answer this I would like to refer to page 108, chapter 2 in the latest IPCC report. There you will find all our "opinions" summarized because there you find a remaining carbon dioxide budget. Right there it says that if we are to have a sixty-seven percent chance of limiting the global temperature rise to below 1.5 degrees, we had on January 1st, 2018, 420 gigatons of carbon dioxide left in our CO2 budget. And of course, that number is much lower today. We emit about 42 gigatons of CO2 every year.

Analysis:  In her address to the National Assembly in Paris, Thunberg cites the 2018 “IPCC report” that outlines a total “remaining carbon dioxide budget” of “420 gigatons” in order to “have a sixty-seven percent chance of limiting the global temperature rise to below 1.5 degrees,” while “we emit about 42 gigatons of CO2 each year.” By citing specific data from a reputable scientific journal, Thunberg appeals to her audience’s logic; the data proves that the only viable option is to limit carbon dioxide emissions.

29. metonymy

Definition: a figure of speech consisting of the use of the name of one thing for that of another of which it is an attribute or with which it is associated

Example:  Margaret Thatcher’s eulogy for Ronald Reagan (2004)

Yet his ideas, so clear, were never simplistic. He saw the many sides of truth. Yes, he warned that the Soviet Union had an insatiable drive for military power and territorial expansion, yet he also sensed that it was being eaten away by systematic failures impossible to reform. Yes, he did not shrink from denouncing Moscow’s evil empire, but he realized that a man of goodwill might nonetheless emerge from its dark corridors.

Analysis:  In her eulogy for United States President Ronald Reagan, Thatcher refers to the Soviet Union as “Moscow’s evil empire.” Her metonymy  explicitly communicates a disdain for the Soviet Union, which establishes common ground between the United States and the United Kingdom, which helps Thatcher strengthen relations with the United States while eulogizing a friend.

Definition: the speed at which a piece of writing flows — use when discussing organization; point out where action/syntax begins to speed up, slow down, is interrupted, etc.

Example:   Notes on ‘Camp’  by Susan Sontag (1964)

1. To start very generally: Camp is a certain mode of aestheticism. It is one way of seeing the world as an aesthetic phenomenon. That way, the way of Camp, is not in terms of beauty, but in terms of the degree of artifice, of stylization.

2. To emphasize style is to slight content, or to introduce an attitude which is neutral with respect to content. It goes without saying that the Camp sensibility is disengaged, depoliticized — or at least apolitical.

Analysis:  Sontag writes Notes on ‘Camp’  as a “series of jottings” rather than in paragraph format in order to mimic the spontaneous and ever-changing nature of Camp. By presenting her notes as a numbered list, Sontag develops a quick, irregular pace  that is more fitting to describe the eccentricities of Camp. Because the notes are presented as a list, the ideas move by quickly, which further mirrors the whimsicality that is so characteristic of Camp.

31. paradox

Definition: apparently self-contradictory statement, the underlying meaning of which is revealed only by careful scrutiny; its purpose is to arrest attention and provoke fresh thought

Example:  “On the Writing of Essays” by Alexander Smith (1881)

He is the frankest, most outspoken of writers; and that very frankness and outspokenness puts the reader off his guard. If you wish to preserve your secret, wrap it up in frankness. The Essays are full of this trick. The frankness is as well simulated as the grape-branches of the Grecian artist which the birds flew towards and pecked. When Montaigne retreats, he does so like a skillful general, leaving his fires burning.

Analysis:  Smith describes Montaigne’s writing style as very frank and outspoken, asserting that “if you wish to preserve your secret, wrap it up in frankness.” Smith’s paradox , although outwardly nonsensical, forces the reader to pause and ruminate on the conflicting ideas, which naturally places emphasis on these ideas. Through his paradox, Smith suggests that an author’s works often contain intimate personal revelations that seem obvious, yet are often overlooked by most readers.

32. parallelism (parallel structure)

Definition: a repetition of sentences using the same grammatical structure emphasizing all aspects of the sentence equally

Example:  “Luckiest Man on the Face of the Earth” by Lou Gherig (1939)

When the New York Giants, a team you would give your right arm to beat, and vice versa, sends you a gift — that's something. When everybody down to the groundskeepers and those boys in white coats remembers you with trophies — that's something. When you have a wonderful mother-in-law who takes sides with you in squabbles with her own daughter — that's something. When you have a father and a mother who work all their lives so you can have an education and build your body — it's a blessing. When you have a wife who has been a tower of strength and shown more courage than you dreamed existed — that's the finest I know.

Analysis:  Gherig presents a series of parallel  sentences to emphasize his gratitude for the life he has lived. Because each sentence follows the same structure, Gherig’s list builds to a climax, which Gherig uses to enumerate his priorities and to emphasize his love for his family. Gherig further emphasizes his appreciation for his family even above his career by shifting from the phrase “that’s something” to describe his wife’s courage as “the finest” he knows. By breaking the pattern in his parallel sentences, Gherig focuses the attention on his family and loved ones, humbly placing his own successes on the back burner.

Definition: appealing to emotion

Example:  Viola Davis’s Women’s March Speech (2018)

I am speaking today not just for the 'Me Toos,' because I was a 'Me Too,' but when I raise my hand, I am aware of all the women who are still in silence. The women who are faceless. The women who don't have the money and don't have the constitution and who don't have the confidence and who don't have the images in our media that gives them a sense of self-worth enough to break their silence that is rooted in the shame of assault and rooted in the stigma of assault.

Analysis:  In her speech at the 2018 Women’s March, Viola Davis recognizes the millions of women who have been silently affected by sexual violence. She describes the women “don’t have the money,” “constitution,” or “confidence,” and those who still struggle with the “shame” and “stigma of assault.” Davis employs anaphora, repeating the phrase “don’t have” to evoke a sense of empathy for these women among the audience. By emphasizing that these victims “don’t have” the resources that many take for granted, Davis sheds light on the cruel reality that many victims still face due to the stigma surrounding sexual assault and women’s rights.

34. polysyndeton

Definition: the use of many conjunctions has the effect of slowing the pace or emphasizing the numerous words or clauses

Example:  “After the Storm” by Ernest Hemingway (1932)

I said, “Who killed him?” and he said, “I don’t know who killed him but he’s dead all right,” and it was dark and there was water standing in the street and no lights and windows broke and boats all up in the town and trees blown down and everything all blown and I got a skiff and went out and found my boat where I had her inside Mango Key and she was all right only she was full of water.

Analysis:  After learning of the murder, the narrator describes as “dark and there was water standing in the street and no lights and windows broke and boats all up in the town,” repeating the conjunction “and.” Hemingway employs

polysyndeton  to illustrate the narrator’s shock and panic following the murder. By inserting “and” between each phrase, Hemingway slows down the pace of the sentence, conveying the sense of the narrator’s surroundings moving in slow motion after hearing the news.

35. rhetorical question

Definition: a question presented by the author that is not meant to be answered

Example:  Clare de Booth Luce’s Speech at the Women’s National Press Club (1960)

For what is good journalism all about? On a working, finite level it is the effort to achieve illuminating candor in print and to strip away cant. It is the effort to do this not only in matters of state, diplomacy, and politics but also in every smaller aspect of life that touches the public interest or engages proper public curiosity.

Analysis:  In her speech at the Women’s National Press Club, de Booth asks the rhetorical question : “For what is good journalism all about?” in order to signal a shift in tone as she moves to describe the purpose of “good journalism.” By asking the audience a question, she invites them to consider their own motivations as journalists as she explains her own belief that “good journalism” is “the effort to achieve illuminating candor in print.” Rather than simply speaking about her views on journalism, de Booth expertly inserts a rhetorical question in order to evoke a moment of wonder and self-reflection in her audience before she answers her own question.

36. stream of consciousness

Definition: a technique that records the thoughts and feelings of a character without regard to logical argument or narrative sequence; reflects all the forces, internal and external, affecting the character's psyche at the moment

Example:  “Ain’t I a Woman” by Sojourner Truth (1851)

That man over there says that women need to be helped into carriages, and lifted over ditches, and to have the best place everywhere. Nobody ever helps me into carriages, or over mud-puddles, or gives me any best place! And ain't I a woman? Look at me! Look at my arm! I have ploughed and planted, and gathered into barns, and no man could head me! And ain't I a woman? I could work as much and eat as much as a man — when I could get it — and bear the lash as well! And ain't I a woman? I have borne thirteen children, and seen most all sold off to slavery, and when I cried out with my mother's grief, none but Jesus heard me! And ain't I a woman?

Analysis:  In 1851, Sojourner Truth gave a moving speech at the Women’s Rights Convention without preparation. Truth’s stream of consciousness  approach to the speech allows her to directly address her audience, beginning by mentioning “that man over there” and refuting his beliefs that women are fragile. Truth then moves to note that she has “ploughed and planted” more successfully than men, and she moves to the fact that she can “work as much and eat as much as a man.” She shifts yet again to recount that she has “borne thirteen children” and that “none but Jesus” heard her cry with her “mother’s grief” when they were sold to slavery. Albeit slightly messy, Truth’s lack of structure is effective because it reflects the never-ending struggles that African American women faced. When the injustices seemed to cease, another injustice would arise in a never-ending cycle of oppression. Truth’s speech thus resonated with many other women who had experienced the same struggles, and Truth became a powerful voice in the fight racial and gender equality.

37. synecdoche

Definition: the rhetorical substitution of a part for the whole

Example:  “Falling Down is Part of Growing Up” by Henry Petroski (1985)

We are transported across impromptu bridges of arms thrown up without plans or blueprints between mother and aunt, between neighbor and father, between brother and sister — none of whom is a registered structural engineer. We come to Mama and to Papa eventually to forget our scare reflex and we learn to trust the beams and girders and columns of their arms and our cribs.

Analysis:  Petroski refers to a child’s parents and crib as “beams and girders and columns” that the child must trust, emphasizing the structural aspect of a young child’s support system. Instead of referring to the parents and crib as a whole, Petroski uses synecdoche  to strip away the sentimental connotations associated with a mother’s arms and a baby’s crib, highlighting only the “beams and girders and columns” that prevent the child from falling and returning to his novel’s central topic of engineering.

Definition: the structure of sentences and/or phrases

We have waited for more than 340 years for our constitutional and God-given rights. The nations of Asia and Africa are moving with jetlike speed toward gaining political independence, but we still creep at horse and buggy pace toward gaining a cup of coffee at a lunch counter. Perhaps it is easy for those who have never felt the stinging darts of segregation to say, "Wait." But when you have seen vicious mobs lynch your mothers and fathers at will and drown your sisters and brothers at whim; when you have seen hate-filled policemen curse, kick and even kill your black brothers and sisters; when you see the vast majority of your twenty million Negro brothers smothering in an airtight cage of poverty in the midst of an affluent society; when you suddenly find your tongue twisted and your speech stammering as you seek to explain to your six-year-old daughter why she can't go to the public amusement park that has just been advertised on television, and see tears welling up in her eyes when she is told that Funtown is closed to colored children, and see ominous clouds of inferiority beginning to form in her little mental sky, and see her beginning to distort her personality by developing an unconscious bitterness toward white people; when you have to concoct an answer for a five-year-old son who is asking: "Daddy, why do white people treat colored people so mean?"; when you take a cross-country drive and find it necessary to sleep night after night in the uncomfortable corners of your automobile because no motel will accept you; when you are humiliated day in and day out by nagging signs reading "white" and "colored"; when your first name becomes "nigger," your middle name becomes "boy" (however old you are) and your last name becomes "John," and your wife and mother are never given the respected title "Mrs."; when you are harried by day and haunted by night by the fact that you are a Negro, living constantly at tiptoe stance, never quite knowing what to expect next, and are plagued with inner fears and outer resentments; when you are forever fighting a degenerating sense of "nobodiness" — then you will understand why we find it difficult to wait.

Analysis:  In his “Letter from a Birmingham Jail,” King addresses those who instruct him to “wait” for racial equality by describing the “stinging pain of segregation” as seeing “vicious mobs lynch your mothers and fathers,” seeing “hate-filled policemen curse, kick, and even kill your brothers and sisters,” and seeing the “tears welling up” in your six-year-old daughter’s eyes when “she is told that Funtown is closed to colored children,” among a host of other horrific images. Rather than using several shorter sentences to describe segregation, King uses a single sentence, separated by numerous semicolons. King’s choice of syntax  mirrors the never-ending reach of segregation and racial inequality. While the sentence consists of a string of short images, it pauses on a longer phrase in which King describes finding his “tongue-twisted” as he explains to his “six-year-old daughter why she can't go to the public amusement park that has just been advertised on television,” and seeing “tears welling up in her eyes when she is told that Funtown is closed to colored children” while he watches the “ominous clouds of inferiority beginning to form in her little mental sky” and her “distort her personality by developing an unconscious bitterness toward white people.” By making this phrase significantly longer than his other images, King allows the reader to pause and ruminate on the idea of a young girl losing her innocence to an unjust world. King appeals to the reader’s emotions as he conveys such a heartbreaking image.

Definition: a statement of purpose, intent, or main idea in a literary work

Example:   Notes on ‘Camp’  by Susan Sontag

58. The ultimate Camp statement: it's good because  it's awful . . . Of course, one can't always say that. Only under certain conditions, those which I've tried to sketch in these notes.

Analysis:  Sontag places her thesis  at the end of her Notes on ‘Camp’ , which allows her to summarize her list and to assert that Camp is “good because  it’s awful.” Sontag concludes the notes by referencing her sporadic list of musings regarding Camp as a whole and declaring them the “conditions” under which Camp can be both good and awful.

the use of stylistic devices that reveal an author’s attitude towards a subject

Example:  “If Black English Isn’t a Language, Then Tell Me, What Is?” by James Baldwin (1979)

I say that the present skirmish is rooted in American history, and it is. Black English is the creation of the black diaspora. Blacks came to the United States chained to each other, but from different tribes: Neither could speak the other's language. If two black people, at that bitter hour of the world's history, had been able to speak to each other, the institution of chattel slavery could never have lasted as long as it did. Subsequently, the slave was given, under the eye, and the gun, of his master, Congo Square, and the Bible–or in other words, and under these conditions, the slave began the formation of the black church, and it is within this unprecedented tabernacle that black English began to be formed. This was not, merely, as in the European example, the adoption of a foreign tongue, but an alchemy that transformed ancient elements into a new language: A language comes into existence by means of brutal necessity, and the rules of the language are dictated by what the language must convey.

Analysis:  Baldwin adopts a formal, academic tone , assessing the development of “Black English” through a historical lens. Baldwin concludes that “Black English is the creation of the black diaspora” as “an alchemy that transformed ancient elements into a new language.” By using academic diction, Baldwin approaches the development of Black English not as a cultural or social issue, but simply as a historical phenomenon that should be studied objectively, which allows him to persuade his readers that Black English should be considered a distinct language.

👉 Play Kahoot with AP Lang teacher Kathryn Howard as she recaps rhetorical strategies and devices!  

One last disclaimer: Fiveable is an educational company without political or religious affiliations and it neither endorses nor opposes any views expressed in the above passages. There you go! When looking at each device and its corresponding example, think of ways and reasons authors integrate these rhetorical devices, styles, and terms into their writing! Thinking that much ahead will pay off when you write the Rhetorical Analysis essay in May! 😄

Fiveable

Stay Connected

© 2024 Fiveable Inc. All rights reserved.

AP® and SAT® are trademarks registered by the College Board, which is not affiliated with, and does not endorse this website.

Articles Hub  | Return to Main page

Vector illustration of an educational workspace with textbooks, a clipboard, and a digital test icon.

Understanding AP English Exam Fundamentals: Tips and Strategies for Success

Earning a high score on an Advanced Placement ® (AP ® ) exam can catapult you to college admissions success. Your AP exam success will look good on your college application, can positively affect your GPA, and can even earn you college credit!

In this article, we’ll delve into the fundamentals and structure of the AP English exams. Understanding the nuances of both AP Language and Composition (Lang) and AP Literature and Composition (Lit), and common key features, including scoring system and effective testing strategies, is essential for student success.

Scoring System

AP exams are scored on a scale from one to five, with five being the highest. The scoring breakdown is as follows:

5: Extremely well-qualified (equivalent to an A or A+)

4: Very well-qualified (ranges from a B to an A-)

3: Qualified (from a C to a B-)

2: Possibly qualified

1: Not recommended

Overview of AP English Exams

Each of the AP English exams focuses on a unique set of themes. As you’ll see, while the content that you’ll need to learn will vary, the strategies that you’ll need to succeed are consistent.

  • AP Language and Composition focuses on analyzing rhetorical arguments through reading and writing texts. You’ll explore topics like rhetorical situation, claims, evidence, reasoning, organization, and style. You will be tasked with evaluating authors’ arguments, identifying purposeful choices, and synthesizing diverse sources to support a point. The exam consists of 45 multiple-choice questions and 3 free-response questions from the following categories: synthesis, rhetorical analysis, and argument. The Lang exam also builds in 15 minutes of reading time for the synthesis free-response section.
  • AP Literature and Composition: AP Lit emphasizes the elements of literature, such as characters, setting, structure, narrator, figurative language, and the use of examples to support arguments. You’ll demonstrate understanding of poetry and prose fiction by dissecting texts to understand authors’ intentions and narrative techniques. The exam consists of 55 multiple-choice questions and 3 free-response questions from the following categories: literary analysis of a given poem, literary analysis of a prose section, and analysis that examines a specific concept, issue, or element in a work you select, ideally one read in your AP Lit class.

Preparing for Success

To excel on your AP English exams, you’ll need to demonstrate subject-matter expertise—and you’ll also need to deploy effective test-taking strategies specific to these exams.

For your free-response questions, practice makes perfect ! Make sure you are familiar with the definition and structure of each essay type, whether it’s a rhetorical analysis or a literary analysis. Engage in regular writing practice and take your in-class timed writings as an opportunity to improve and receive valuable feedback.

Remember to look at examples of successful student work from prior exam periods— examples of student essays for Lang and examples for Lit are available for free on The College Board’s website.

Learn how to be persuasive . Books like Thank You for Arguing by Jill Heinrichs can help you expand your understanding of persuasion, audience, tone, and open-mindedness.

Familiarize yourself with various literary works , including those outside your required reading list, to bolster comprehension and enhance your ability to support arguments with textual evidence.

Remember to check in with your school for any specific procedures related to AP exams and review your knowledge regularly to maximize your performance on exam day.

Whether tackling an argumentative essay or analyzing a poem, preparation and strategy are paramount. Familiarize yourself with the exam formats, employ effective test-taking strategies, and hone your writing skills to best navigate these AP English exams. You are already on your way to maximizing your AP test scores!

To learn more about effective AP English exam-taking strategies, check out our webinar!

AP ®  and Advanced Placement ®  are trademarks registered by the College Board.

Share on Facebook

Proudly Serving

Data Protection Statement | Privacy Policy | CA Privacy Notice | Terms of Use

©2024 Tutor.com / The Princeton Review. Not Affiliated with Princeton University

Tutor.com is controlled by Primavera Holdings Limited, a firm owned by Chinese nationals with a principal place of business in Hong Kong, China.

IMAGES

  1. How to Write the AP Lang Rhetorical Essay

    rhetorical analysis essay ap lang college board

  2. How To Write A Rhetorical Analysis Essay AP Lang In 6 Steps

    rhetorical analysis essay ap lang college board

  3. Rhetorical Analysis Bundle

    rhetorical analysis essay ap lang college board

  4. AP Language-- Rhetorical devices A-D

    rhetorical analysis essay ap lang college board

  5. Final Rhetorical Analysis Essay

    rhetorical analysis essay ap lang college board

  6. AP Lang. & Comp. Rhetorical Analysis Essay Rubric by Lover of Language Arts

    rhetorical analysis essay ap lang college board

VIDEO

  1. Rhetorical Analysis Essay Tutorial

  2. Rhetorical Analysis Overview for AP Lang Students

  3. Revisions and Peer-Edits in AP Lang

  4. AP Lang

  5. How to Plan a Rhetorical Analysis Unit

  6. I Wrote an Ebook!

COMMENTS

  1. AP English Language and Composition Past Exam Questions

    Download free-response questions from past exams along with scoring guidelines, sample responses from exam takers, and scoring distributions. If you are using assistive technology and need help accessing these PDFs in another format, contact Services for Students with Disabilities at 212-713-8333 or by email at [email protected]. Note ...

  2. How to Write the AP Lang Rhetorical Analysis Essay (With Example)

    Her story "The Astronaut" won the 2018 Shirley Jackson Award for short fiction and received a "Distinguished Stories" mention in the 2019 Best American Short Stories anthology. How to write the AP Lang rhetorical analysis essay. We look at a AP lang rhetorical analysis essay example and explore do's and don'ts.

  3. PDF AP English Language and Composition

    1 − Essays earning a score of 1 meet the criteria for the score of 2 but are undeveloped, especially simplistic in their explanation, or weak in their control of language. 0 - Indicates an off-topic response, one that merely repeats the prompt, an entirely crossed-out response, a drawing, or a response in a language other than English.

  4. AP Lang

    Timing. You have 40 minutes to complete the rhetorical analysis essay for AP Lang: 12 minutes: Read the text and plan out your essay. (TOBI) 6 minutes: Write your introduction paragraph. 18 minutes: Write 2-3 body paragraphs. 2 minutes: Write a quick conclusion. 2 minutes: Proofread and revise your essay.

  5. How to Write the AP Lang Rhetorical Essay

    Tips for Writing the AP Lang Rhetorical Essay. 1. Outline Your Essay Before Writing. One of the most important parts of the AP Lang essays is structuring your essay so that it makes sense to the reader. This is just as important as having good content. For this essay in particular, you'll want to read the passage first and write a brief ...

  6. AP Lang

    AP Lang Rhetorical Analysis Essay Practice. ... College Board says that you should "Explain how multiple rhetorical choices in the passage contribute to the writer's argument, purpose, or message" and while it does provide a caveat that "the response may observe multiple instances of the same rhetorical choice if each instance further ...

  7. AP English Language and Composition

    AP English Language and Composition Course and Exam Description. This is the core document for this course. Unit guides clearly lay out the course content and skills and recommend sequencing and pacing for them throughout the year. The CED was updated in the summer of 2020 to include scoring guidelines for the example questions.

  8. PDF AP English Language and Composition Question 2: Rhetorical Analysis

    AP English Language and Composition Question 2: Rhetorical Analysis (2019) Sample Student Responses 1 The student responses in this packet were selected from the 2019 Reading and have been rescored using the new rubrics for 2020. Commentaries for each sample are provided in a separate document.

  9. PDF ap06 english lang student samples

    adequately. Essays earning a score of 6 respond to the prompt . They adequately analyze the rhetorical strategies Hazlitt uses to develop his position about money. These essays may refer to the passage explicitly or implicitly. The writing may contain lapses in diction or syntax, but generally the prose is clear.

  10. PDF AP English Language and Composition

    Rhetorical Analysis 6 points On February 27, 2013, while in office, former president Barack Obama delivered the following address dedicating the Rosa Parks st atue in the National Statuary Hall of the United States Capitol building.

  11. PDF AP® ENGLISH LANGUAGE AND COMPOSITION

    2010 The College Board. Visit the College Board on the Web: www.collegeboard.com. AP® ENGLISH LANGUAGE AND COMPOSITION 2010 SCORING GUIDELINES. Question 2 (continued) 3 Essays earning a score of 3 meet the criteria for a score of 4 but demonstrate less success in analyzing Banneker's use of rhetorical strategies to argue against slavery ...

  12. How to master the AP English Language and Composition rhetorical

    Use precise, concise language: Clearly articulate your analysis in a direct, concise manner. Use specific language and active verbs when analyzing the author's effectiveness. 6. Practice with past prompts: To get better at rhetorical analysis essays, practice by reviewing past AP exam prompts and analyzing various texts.

  13. AP Lang rhetorical analysis essay examples?

    1. College Board resources: The College Board, which oversees the AP program, provides sample essays and scoring guidelines on their website. You can find past exam questions and sample responses for AP Lang, including rhetorical analysis essays, in their "Free-Response Questions" section.

  14. 3 AP® English Language Rhetorical Essay Strategies

    The essay becomes clear, assertive, and easy to follow for the examiners. Follow this rhetorical essay strategy and you are even closer to getting that 5 on the exam. Rhetorical Essay Strategy #3: LORA. As you are looking at your AP® English Language rhetorical essay prompt and passage it is important to remember the mnemonic device, LORA.

  15. AP Lang Analysis Essay

    The AP Lang analysis essay, also known as the rhetorical analysis essay, requires you to analyze a text (usually a speech or an article) and explain how the author uses various rhetorical strategies to achieve their purpose. ... For additional resources, consider checking out the College Board's AP Lang resources (https://apcentral.collegeboard ...

  16. PDF AP® ENGLISH LANGUAGE AND COMPOSITION

    Sample: 2A Score: 8. This essay effectively identifies and analyzes three of Cesar Chavez's rhetorical choices — striking diction, juxtaposition, and appeals to reader's fundamental moral beliefs — to argue that "nonviolence is the best and most moral way to bring change.". Providing convincing evidence and analysis (for example ...

  17. PDF AP® ENGLISH LANGUAGE AND COMPOSITION

    1 - Essays earning a score of 1 meet the criteria for the score of 2 but are undeveloped, especially simplistic in their explanation, or weak in their control of language. 0 - Indicates an off-topic response, one that merely repeats the prompt, an entirely crossed-out response, a drawing, or a response in a language other than English.

  18. AP Lang

    Yes, the rhetorical analysis essay is an argument essay just like the other two. You aren't required to use rhetorical vocabulary in your essays at all — in fact, it's probably better if you don't. If you force the vocabulary into your essay, you risk sounding clunky, and the vocabulary almost always leads you to switch to passive voice.

  19. Succeed in AP English: Essential Exam Preparation Tips

    Scoring System. AP exams are scored on a scale from one to five, with five being the highest. The scoring breakdown is as follows: 5: Extremely well-qualified (equivalent to an A or A+) 4: Very well-qualified (ranges from a B to an A-) 3: Qualified (from a C to a B-) 2: Possibly qualified. 1: Not recommended.